Showing $gcd(2^m-1,2^n+1)=1$












7












$begingroup$


A student of mine has been self-studying some elementary number theory. She came by my office today and asked if I had any hints on how to prove the statement




If $m$ is odd then $gcd(2^m-1,2^n+1)=1$.




It's been a while since I took number theory and I'm not sure what to do. She said she is learning about congruences, primitive roots, and power residues. She has not taken any group theory.










share|cite|improve this question











$endgroup$

















    7












    $begingroup$


    A student of mine has been self-studying some elementary number theory. She came by my office today and asked if I had any hints on how to prove the statement




    If $m$ is odd then $gcd(2^m-1,2^n+1)=1$.




    It's been a while since I took number theory and I'm not sure what to do. She said she is learning about congruences, primitive roots, and power residues. She has not taken any group theory.










    share|cite|improve this question











    $endgroup$















      7












      7








      7


      4



      $begingroup$


      A student of mine has been self-studying some elementary number theory. She came by my office today and asked if I had any hints on how to prove the statement




      If $m$ is odd then $gcd(2^m-1,2^n+1)=1$.




      It's been a while since I took number theory and I'm not sure what to do. She said she is learning about congruences, primitive roots, and power residues. She has not taken any group theory.










      share|cite|improve this question











      $endgroup$




      A student of mine has been self-studying some elementary number theory. She came by my office today and asked if I had any hints on how to prove the statement




      If $m$ is odd then $gcd(2^m-1,2^n+1)=1$.




      It's been a while since I took number theory and I'm not sure what to do. She said she is learning about congruences, primitive roots, and power residues. She has not taken any group theory.







      elementary-number-theory divisibility greatest-common-divisor






      share|cite|improve this question















      share|cite|improve this question













      share|cite|improve this question




      share|cite|improve this question








      edited Feb 16 '16 at 21:37









      Martin Sleziak

      44.7k8117272




      44.7k8117272










      asked Nov 18 '13 at 19:06









      Joe Johnson 126Joe Johnson 126

      13.8k32770




      13.8k32770






















          1 Answer
          1






          active

          oldest

          votes


















          9












          $begingroup$

          If an odd prime $p$ divides $2^n+1$, then the order of $2$ modulo $p$ is even (it is a divisor of $2n$, but not of $n$). If an odd prime $q$ divides $2^m-1$ with $m$ odd, then the order of $2$ modulo $q$ is odd (it is a divisor of $m$). Hence $p neq q$. Since $2^m - 1$ is odd for $m > 0$, in particular all odd $m$, the greatest common divisor cannot be even. So no prime divides both, $2^n+1$ and $2^m-1$.



          Alternatively, we can use



          $$gcd (2^t-1, 2^u-1) = 2^{gcd (t,u)}-1tag{1}$$



          to conclude



          $$gcd (2^m-1, 2^{2n}-1) = 2^{gcd(m,2n)}-1.$$



          But since $m$ is odd, we have $gcd (m,2n) = gcd(m,n)$, and hence



          $$2^{gcd(m,2n)}-1 mid 2^n-1,$$



          which, since



          $$gcd(2^n-1,2^n+1) = gcd(2^n-1,2) mid 2$$



          and $2^{gcd(m,2n)}-1$ is odd, implies $gcd (2^{gcd(m,2n)}-1,2^n+1) = 1$ and hence $gcd(2^m-1,2^n+1) = 1$.



          To see $(1)$, write $u = qcdot t + r$ with $0 leqslant r < t$, and



          $$2^u-1 = 2^rleft(2^{qcdot t}-1right) + left(2^r-1right),$$



          which, since $2^t-1 mid (2^t)^q-1$, yields



          $$gcd(2^t-1,2^u-1) = gcd(2^t-1,2^r-1),$$



          and continuing the Euclidean algorithm for the exponents finally yields $(1)$.






          share|cite|improve this answer











          $endgroup$













            Your Answer





            StackExchange.ifUsing("editor", function () {
            return StackExchange.using("mathjaxEditing", function () {
            StackExchange.MarkdownEditor.creationCallbacks.add(function (editor, postfix) {
            StackExchange.mathjaxEditing.prepareWmdForMathJax(editor, postfix, [["$", "$"], ["\\(","\\)"]]);
            });
            });
            }, "mathjax-editing");

            StackExchange.ready(function() {
            var channelOptions = {
            tags: "".split(" "),
            id: "69"
            };
            initTagRenderer("".split(" "), "".split(" "), channelOptions);

            StackExchange.using("externalEditor", function() {
            // Have to fire editor after snippets, if snippets enabled
            if (StackExchange.settings.snippets.snippetsEnabled) {
            StackExchange.using("snippets", function() {
            createEditor();
            });
            }
            else {
            createEditor();
            }
            });

            function createEditor() {
            StackExchange.prepareEditor({
            heartbeatType: 'answer',
            autoActivateHeartbeat: false,
            convertImagesToLinks: true,
            noModals: true,
            showLowRepImageUploadWarning: true,
            reputationToPostImages: 10,
            bindNavPrevention: true,
            postfix: "",
            imageUploader: {
            brandingHtml: "Powered by u003ca class="icon-imgur-white" href="https://imgur.com/"u003eu003c/au003e",
            contentPolicyHtml: "User contributions licensed under u003ca href="https://creativecommons.org/licenses/by-sa/3.0/"u003ecc by-sa 3.0 with attribution requiredu003c/au003e u003ca href="https://stackoverflow.com/legal/content-policy"u003e(content policy)u003c/au003e",
            allowUrls: true
            },
            noCode: true, onDemand: true,
            discardSelector: ".discard-answer"
            ,immediatelyShowMarkdownHelp:true
            });


            }
            });














            draft saved

            draft discarded


















            StackExchange.ready(
            function () {
            StackExchange.openid.initPostLogin('.new-post-login', 'https%3a%2f%2fmath.stackexchange.com%2fquestions%2f572248%2fshowing-gcd2m-1-2n1-1%23new-answer', 'question_page');
            }
            );

            Post as a guest















            Required, but never shown

























            1 Answer
            1






            active

            oldest

            votes








            1 Answer
            1






            active

            oldest

            votes









            active

            oldest

            votes






            active

            oldest

            votes









            9












            $begingroup$

            If an odd prime $p$ divides $2^n+1$, then the order of $2$ modulo $p$ is even (it is a divisor of $2n$, but not of $n$). If an odd prime $q$ divides $2^m-1$ with $m$ odd, then the order of $2$ modulo $q$ is odd (it is a divisor of $m$). Hence $p neq q$. Since $2^m - 1$ is odd for $m > 0$, in particular all odd $m$, the greatest common divisor cannot be even. So no prime divides both, $2^n+1$ and $2^m-1$.



            Alternatively, we can use



            $$gcd (2^t-1, 2^u-1) = 2^{gcd (t,u)}-1tag{1}$$



            to conclude



            $$gcd (2^m-1, 2^{2n}-1) = 2^{gcd(m,2n)}-1.$$



            But since $m$ is odd, we have $gcd (m,2n) = gcd(m,n)$, and hence



            $$2^{gcd(m,2n)}-1 mid 2^n-1,$$



            which, since



            $$gcd(2^n-1,2^n+1) = gcd(2^n-1,2) mid 2$$



            and $2^{gcd(m,2n)}-1$ is odd, implies $gcd (2^{gcd(m,2n)}-1,2^n+1) = 1$ and hence $gcd(2^m-1,2^n+1) = 1$.



            To see $(1)$, write $u = qcdot t + r$ with $0 leqslant r < t$, and



            $$2^u-1 = 2^rleft(2^{qcdot t}-1right) + left(2^r-1right),$$



            which, since $2^t-1 mid (2^t)^q-1$, yields



            $$gcd(2^t-1,2^u-1) = gcd(2^t-1,2^r-1),$$



            and continuing the Euclidean algorithm for the exponents finally yields $(1)$.






            share|cite|improve this answer











            $endgroup$


















              9












              $begingroup$

              If an odd prime $p$ divides $2^n+1$, then the order of $2$ modulo $p$ is even (it is a divisor of $2n$, but not of $n$). If an odd prime $q$ divides $2^m-1$ with $m$ odd, then the order of $2$ modulo $q$ is odd (it is a divisor of $m$). Hence $p neq q$. Since $2^m - 1$ is odd for $m > 0$, in particular all odd $m$, the greatest common divisor cannot be even. So no prime divides both, $2^n+1$ and $2^m-1$.



              Alternatively, we can use



              $$gcd (2^t-1, 2^u-1) = 2^{gcd (t,u)}-1tag{1}$$



              to conclude



              $$gcd (2^m-1, 2^{2n}-1) = 2^{gcd(m,2n)}-1.$$



              But since $m$ is odd, we have $gcd (m,2n) = gcd(m,n)$, and hence



              $$2^{gcd(m,2n)}-1 mid 2^n-1,$$



              which, since



              $$gcd(2^n-1,2^n+1) = gcd(2^n-1,2) mid 2$$



              and $2^{gcd(m,2n)}-1$ is odd, implies $gcd (2^{gcd(m,2n)}-1,2^n+1) = 1$ and hence $gcd(2^m-1,2^n+1) = 1$.



              To see $(1)$, write $u = qcdot t + r$ with $0 leqslant r < t$, and



              $$2^u-1 = 2^rleft(2^{qcdot t}-1right) + left(2^r-1right),$$



              which, since $2^t-1 mid (2^t)^q-1$, yields



              $$gcd(2^t-1,2^u-1) = gcd(2^t-1,2^r-1),$$



              and continuing the Euclidean algorithm for the exponents finally yields $(1)$.






              share|cite|improve this answer











              $endgroup$
















                9












                9








                9





                $begingroup$

                If an odd prime $p$ divides $2^n+1$, then the order of $2$ modulo $p$ is even (it is a divisor of $2n$, but not of $n$). If an odd prime $q$ divides $2^m-1$ with $m$ odd, then the order of $2$ modulo $q$ is odd (it is a divisor of $m$). Hence $p neq q$. Since $2^m - 1$ is odd for $m > 0$, in particular all odd $m$, the greatest common divisor cannot be even. So no prime divides both, $2^n+1$ and $2^m-1$.



                Alternatively, we can use



                $$gcd (2^t-1, 2^u-1) = 2^{gcd (t,u)}-1tag{1}$$



                to conclude



                $$gcd (2^m-1, 2^{2n}-1) = 2^{gcd(m,2n)}-1.$$



                But since $m$ is odd, we have $gcd (m,2n) = gcd(m,n)$, and hence



                $$2^{gcd(m,2n)}-1 mid 2^n-1,$$



                which, since



                $$gcd(2^n-1,2^n+1) = gcd(2^n-1,2) mid 2$$



                and $2^{gcd(m,2n)}-1$ is odd, implies $gcd (2^{gcd(m,2n)}-1,2^n+1) = 1$ and hence $gcd(2^m-1,2^n+1) = 1$.



                To see $(1)$, write $u = qcdot t + r$ with $0 leqslant r < t$, and



                $$2^u-1 = 2^rleft(2^{qcdot t}-1right) + left(2^r-1right),$$



                which, since $2^t-1 mid (2^t)^q-1$, yields



                $$gcd(2^t-1,2^u-1) = gcd(2^t-1,2^r-1),$$



                and continuing the Euclidean algorithm for the exponents finally yields $(1)$.






                share|cite|improve this answer











                $endgroup$



                If an odd prime $p$ divides $2^n+1$, then the order of $2$ modulo $p$ is even (it is a divisor of $2n$, but not of $n$). If an odd prime $q$ divides $2^m-1$ with $m$ odd, then the order of $2$ modulo $q$ is odd (it is a divisor of $m$). Hence $p neq q$. Since $2^m - 1$ is odd for $m > 0$, in particular all odd $m$, the greatest common divisor cannot be even. So no prime divides both, $2^n+1$ and $2^m-1$.



                Alternatively, we can use



                $$gcd (2^t-1, 2^u-1) = 2^{gcd (t,u)}-1tag{1}$$



                to conclude



                $$gcd (2^m-1, 2^{2n}-1) = 2^{gcd(m,2n)}-1.$$



                But since $m$ is odd, we have $gcd (m,2n) = gcd(m,n)$, and hence



                $$2^{gcd(m,2n)}-1 mid 2^n-1,$$



                which, since



                $$gcd(2^n-1,2^n+1) = gcd(2^n-1,2) mid 2$$



                and $2^{gcd(m,2n)}-1$ is odd, implies $gcd (2^{gcd(m,2n)}-1,2^n+1) = 1$ and hence $gcd(2^m-1,2^n+1) = 1$.



                To see $(1)$, write $u = qcdot t + r$ with $0 leqslant r < t$, and



                $$2^u-1 = 2^rleft(2^{qcdot t}-1right) + left(2^r-1right),$$



                which, since $2^t-1 mid (2^t)^q-1$, yields



                $$gcd(2^t-1,2^u-1) = gcd(2^t-1,2^r-1),$$



                and continuing the Euclidean algorithm for the exponents finally yields $(1)$.







                share|cite|improve this answer














                share|cite|improve this answer



                share|cite|improve this answer








                edited Nov 18 '13 at 22:11

























                answered Nov 18 '13 at 19:09









                Daniel FischerDaniel Fischer

                173k16161283




                173k16161283






























                    draft saved

                    draft discarded




















































                    Thanks for contributing an answer to Mathematics Stack Exchange!


                    • Please be sure to answer the question. Provide details and share your research!

                    But avoid



                    • Asking for help, clarification, or responding to other answers.

                    • Making statements based on opinion; back them up with references or personal experience.


                    Use MathJax to format equations. MathJax reference.


                    To learn more, see our tips on writing great answers.




                    draft saved


                    draft discarded














                    StackExchange.ready(
                    function () {
                    StackExchange.openid.initPostLogin('.new-post-login', 'https%3a%2f%2fmath.stackexchange.com%2fquestions%2f572248%2fshowing-gcd2m-1-2n1-1%23new-answer', 'question_page');
                    }
                    );

                    Post as a guest















                    Required, but never shown





















































                    Required, but never shown














                    Required, but never shown












                    Required, but never shown







                    Required, but never shown

































                    Required, but never shown














                    Required, but never shown












                    Required, but never shown







                    Required, but never shown







                    Popular posts from this blog

                    Can a sorcerer learn a 5th-level spell early by creating spell slots using the Font of Magic feature?

                    Does disintegrating a polymorphed enemy still kill it after the 2018 errata?

                    A Topological Invariant for $pi_3(U(n))$